Practice Questions Flashcards

1
Q
What percentage of the full Social Security benefit will a worker receive if he or she retires 9 months prior to attaining the individual's full Social Security retirement age?
(A) 50 percent of total benefit
(B) 80 percent of total benefit
(C) 95 percent of total benefit
(D) 100 percent of total benefit
A

The answer is (C). The benefit is reduced by 5/9 of one percent for each month prior to the full Social Security retirement age (5/9 of 1% x 9 = 5%). (A), (B), and (D) do not use the correct formula. (Chapter 19)

How well did you know this?
1
Not at all
2
3
4
5
Perfectly
2
Q
An employee exercises nonqualified stock options with an option price of $5 a share and a market price of $10 a share. How much ordinary income does the individual have for each share at the time of exercise?
(A) $ 0
(B) $ 5
(C) $ 10
(D) $ 15
A

The answer is (B). (A), (C), and (D) are incorrect. With a nonqualified stock option, the participant receives ordinary income in the amount of the difference between the market value at the time of exercise and the option price ($10 - $5 = $5). (Chapter 16)

How well did you know this?
1
Not at all
2
3
4
5
Perfectly
3
Q

Which of the following statements concerning the SIMPLE is correct?
(A) To sponsor a SIMPLE, an employer must have 25 or fewer employees.
(B) An employer can sponsor both a SIMPLE and a SEP.
(C) An employer cannot place any restrictions on participant withdrawals.
(D) The deferral limit for a SIMPLE is the same as for a 401(k) plan.

A

The answer is (C). (A) is incorrect because the maximum number of employees an employer can have and still sponsor a SIMPLE is 100, not 25. (B) is incorrect because a SIMPLE sponsor cannot sponsor any other tax-advantaged retirement plan, including qualified plans, SEPs, and 403(b) plans. (D) is incorrect because the maximum employee salary deferral in a SIMPLE is lower than in a 401(k) plan. (Chapter 6)

How well did you know this?
1
Not at all
2
3
4
5
Perfectly
4
Q

Which of the following statements concerning contributions to a SIMPLE is correct?
(A) The employer can elect to provide a 50-cent match for each dollar that the employee elects to defer, as long as the employer matches a salary deferral of up to 6 percent of compensation.
(B) Employee salary deferrals are subject to a nondiscrimination test.
(C) A contribution for all eligible employees in the amount of 2 percent of compensation satisfies the employer-contribution requirement.
(D) The employer has the option to skip making contributions for a specific year.

A

The answer is (C). (A) is incorrect because the employer-matching contribution to a SIMPLE must be a dollar-for-dollar match up to 3 percent that employees elect to defer. (B) is incorrect because employees can make pretax salary deferrals of up to the maximum deferral limit without regard to how much the other employees contribute. (D) is incorrect because the employer must always contribute either a matching contribution or a nonelective contribution. (Chapter 6)

How well did you know this?
1
Not at all
2
3
4
5
Perfectly
5
Q

Which of the following statements concerning employee contributions to a qualified plan (other than to a 401(k) plan) is correct?
(A) Employee contributions to a plan must comply with the top-heavy vesting schedule.
(B) Voluntary employee contributions must satisfy a nondiscrimination test.
(C) Contributions of up to 50 percent of pay can be made if a plan allows voluntary employee contributions.
(D) Voluntary contributions are not counted as annual additions under the limit of Code Sec. 415c. (C).

A

The answer is (B). Voluntary contributions must satisfy the ACP test. Essentially, this means that highly compensated employees cannot make such contributions unless nonhighly compensated employees contribute to the plan. (A) is incorrect because voluntary contributions must be fully vested at all times. (C) is incorrect because it is a nonexistent requirement. (D) is incorrect because voluntary contributions are treated as annual additions. (Chapter 8)

How well did you know this?
1
Not at all
2
3
4
5
Perfectly
6
Q

The systematic withdrawal strategy to retirement income
(A) relies heavily on annuities, bonds, and guarantee income streams to create a systematic income stream to meet basic expenses every year.
(B) often relies on market investments like stocks in order to generate investment growth to help meet retirement income needs.
(C) does not allow for a retiree to be invested in bonds, CDs, or annuities due to their low yields.
(D) cannot provide for flexible spending behaviors of the client nor can it adjust the income to deal with inflation.

A

(B) is correct
(A) is incorrect because this is describing the flooring approach not the systematic withdrawal approach. (C) is incorrect because the systematic withdrawal approach takes an agnostic view of products, and will utilize all of these investment options if it can help improve total returns for the portfolio through diversification and improved safety of investments. (D) is incorrect because systematic withdrawal strategies can be built to increase income over time to handle inflation or variable spending needs.

How well did you know this?
1
Not at all
2
3
4
5
Perfectly
7
Q
If a defined-benefit plan covered by the PBGC has sufficient assets to pay the present value of accrued benefits, the plan qualifies for a
(A) standard termination
(B) distress termination
(C) abandoned plan termination
(D) simple termination
A

standard termination

How well did you know this?
1
Not at all
2
3
4
5
Perfectly
8
Q

Which of the following statements concerning a stock option program is correct?
(A) The employer does not ever get a deduction with a nonqualified stock option program.
(B) A nonqualified stock option program must be available for all employees.
(C) With an incentive stock option (ISO) program, the participants must meet certain holding period requirements to take full advantage of the special tax rules.
(D) There are never any income tax consequences at the time participants sell stock within a nonqualifed stock option program.

A

The answer is (C). (A) is incorrect because, at the time of exercise, the employer gets a deduction in the amount that the participant has in ordinary income. (B) is incorrect because, like other programs of executive compensation, the program can be only for a small group of executives. (D) is incorrect because, at the time of the sale, the participant has long-term capital gain if the sale price exceeds the market value at the time of exercise. (Chapter 16)

How well did you know this?
1
Not at all
2
3
4
5
Perfectly
9
Q

Which of the following statements concerning defined-contribution plans with contribution formulas that are integrated with Social Security is correct?
(A) Only money-purchase pension plans can have a formula that integrates with Social Security.
(B) A plan with a formula that integrates with Social Security must be tested every year to ensure the plan does not violate the 401a. (A)(4) requirements.
(C) The maximum integration level is covered compensation.
(D) If the integration level is the taxable wage base and employees receive a contribution of 3 percent of total compensation, an additional 3 percent can be contributed for compensation in excess of the taxable wage base.

A

The answer is (D). (A) is incorrect because all types of defined-contribution plans can be integrated with Social Security. (B) is incorrect because properly integrated formulas satisfy a design safe harbor. This means that nondiscrimination tests do not have to be performed each year. (C) is incorrect because the maximum integration level in a defined-contribution plan is the current year’s taxable wage base. Covered compensation is typically the integration level in a defined-benefit plan. (Chapter 8)

How well did you know this?
1
Not at all
2
3
4
5
Perfectly
10
Q

Which of the following statements concerning the Code Sec. 401a. (A)(4) nondiscrimination rules that apply to qualified plans is correct?
(A) The regulations require that all plans perform a mathematical test each year.
(B) The regulations confirm that a plan satisfies the rules if highly compensated employees receive contributions that are no more than two times the rate of contribution for nonhighly compensated employees.
(C) There are only subjective “fact and circumstances” tests for determining whether a plan’s benefit or contribution formula satisfies the nondiscrimination rules.
(D) A plan can show that it satisfies the nondiscrimination rules by demonstrating that either benefits or contributions are nondiscriminatory.

A

The answer is (D). This is a correct statement of the rules. (A) is incorrect because a plan that satisfies one of the design safe-harbors does not have to do any other testing. (B) is incorrect because there is no such rule. The rules do say that if contributions constitute a level percentage of compensation for all employees, then the plan will be in compliance. (C) is incorrect because there are clear objective tests under the regulations for determining whether a plan satisfies the rules. (Chapter 8)

How well did you know this?
1
Not at all
2
3
4
5
Perfectly
11
Q

For taxpayers under age 70½ who are active participants in an employer-maintained qualified plan, which of the following statements concerning annual deductible contributions to an individual retirement account (IRA) in 2017 is correct?

(A) For unmarried taxpayers, a full IRA deduction is allowed for adjusted gross income of up to $62,000.
(B) For married taxpayers filing jointly, a full IRA deduction is allowed for adjusted gross income of up to $119,000.
(C) For unmarried taxpayers, a reduced deduction is allowed for adjusted gross income of up to $150,000.
(D) These taxpayers are ineligible to make any tax-deductible contributions to an IRA.

A

The answer is (A). Table 17-1 in the textbook summarize all of the applicable rules. (B), (C), and (D) are incorrect because they do not correctly state the IRA phaseout limits. (Chapter 17)

How well did you know this?
1
Not at all
2
3
4
5
Perfectly
12
Q

Esther is a participant in her employer’s profit-sharing plan. Esther attained age 70 on October 1, 2014. She is not a 5 percent owner and she retires on January 15, 2017. Her required beginning date is

(A) April 1, 2015
(B) April 1, 2016
(C) April 1, 2017
(D) April 1, 2018

A

The answer is (D). Because this is a qualified plan, Esther is not a 5 percent owner, and because she continues working beyond age 70½, the required beginning date is the April 1 following the calendar year in which she retires. (Chapter 24)

How well did you know this?
1
Not at all
2
3
4
5
Perfectly
13
Q

Which of the following statements concerning participant loans from qualified retirement plans is correct?
(A) A qualified plan must include a participant loan provision in one form or another.
(B) Nonhighly compensated participants typically are charged below-market interest rates.
(C) Loans frequently are secured by using a participant’s account balance as security.
(D) Loans can be limited only to the owners of the company.

A

The answer is (C). Securing the loan with the account balance is a good idea because it both satisfies the DOL’s regulations and simplifies the administration of the loan. (A) is incorrect because a plan need not include a loan provision. (B) is incorrect because the plan is required to charge a market rate of interest. (D) is incorrect because a loan program must be available to all participants. (Chapter 9)

How well did you know this?
1
Not at all
2
3
4
5
Perfectly
14
Q

Which of the following statements concerning Code Sec. 409A that applies to nonqualified deferred-compensation plans is correct?
(A) Deferral amounts not subject to a substantial risk of forfeiture that do not comply with the distribution provisions are subject to a 100 percent penalty tax.
(B) Payments to meet the requirements of a domestic relations order do not violate the acceleration of benefit provisions.
(C) A distribution to pay for a child’s college education is likely to be considered an unforeseeable emergency.
(D) A participant will generally be able to make a benefit election at the time distributions are to be made from the plan.

A

The answer is (B). This is an exception to the general rule that benefits cannot be accelerated. (A) is incorrect because the penalty tax is 20 percent. (C) is incorrect because regulations specifically identify this as a situation that is not an unforeseeable emergency. (D) is incorrect because elections have to be made at the time of the deferral election, not the time of distribution. (Chapter 15)

How well did you know this?
1
Not at all
2
3
4
5
Perfectly
15
Q

Which of the following statements concerning the reversion of assets in a qualified plan is correct?
(A) There is a 50 percent excise tax if the entire amount of the excess assets in a defined-benefit plan is reverted to the employer.
(B) A reversion can occur in a defined-contribution plan if assets exceed promised assets.
(C) A defined-benefit plan can be amended at the time of termination to allow for a reversion of plan assets.
(D) There is no excise tax if 20 percent of the excess assets is shared with participants in a defined-benefit plan.

A

The answer is (A). The excise tax is a prohibitive 50 percent unless a portion of the excess is shared with participants. (B) is incorrect because a defined-contribution plan does not have excess assets. All assets are allocated among the participants. (C) is incorrect because a defined-benefit plan must provide for a reversion at the time the plan is drafted. It cannot be amended later to allow a reversion. (D) is incorrect because the tax does not go away if a portion of the excess is shared with participants. The tax is reduced to 20 percent. (Chapter 14)

How well did you know this?
1
Not at all
2
3
4
5
Perfectly
16
Q

As far as profit-sharing plans are concerned, the IRS will recognize an involuntary termination under which of the following circumstances?
(A) The minimum-funding standards have not been satisfied.
(B) The plan contributions are not substantial and recurring.
(C) The long-run liability of the company to the PBGC is expected to increase unreasonably.
(D) A substantial owner has terminated with a plan distribution of over $10,000 and the plan is left with unfunded vested liabilities.

A

The answer is (B). In the case of profit-sharing-type plans, failure to make substantial and recurring contributions results in an involuntary plan termination as of the time contributions cease. Participants become fully vested in their benefits as of the date of the involuntary plan termination. (A) and (C) are incorrect because profit-sharing plans are not subject to actuarial funding. (D) is incorrect because other restrictions apply. (Chapter 14)

How well did you know this?
1
Not at all
2
3
4
5
Perfectly
17
Q
Jim Rock, aged 65, is retired and annually receives a fully taxable $29,000 pension income and $9,000 in Social Security benefits. Jim has no other sources of retirement income and files jointly with his wife. How much of Jim's Social Security benefits will be subject to federal income taxation given his provisional income?
(A) $0
(B) $ 750
(C) $ 1,500
(D) $ 9,000
A

The answer is (B). Because Jim Rock’s adjusted gross income ($29,000) plus one-half of his Social Security income ($9,000/2 = $4,500) exceeds $32,000 (but does not exceed $44,000), as a married taxpayer, he will have to include $750, or one-half of the excess over the $32,000 threshold, as taxable income ($33,500 - $32,000 = 1,500/2 = $750). (A), (C), and (D) do not use the correct formula. (Chapter 19)

How well did you know this?
1
Not at all
2
3
4
5
Perfectly
18
Q

Which of the following statements concerning a Sec. 423 employee stock purchase plan is correct?
(A) The effective discount can be substantially higher than 15 percent off the stock’s price.
(B) The plan can be limited to a few highly compensated executives.
(C) Participants pay taxes at the time the stock is purchased.
(D) Like nonqualified stock options, the entire value of the discount is taxed as ordinary income.

A

The answer is (A). If the price of the stock rises over the purchase period and the price is 15 percent off the market price at the beginning of the period, the discount can vastly exceed 15 percent of the market price of the stock at the time of the purchase. (B) is incorrect because these plans must cover most full-time employees. (C) is incorrect because taxes are not paid until the stock is sold. (D) is incorrect because only a portion of the value is taxed as ordinary income. (Chapter 16)

How well did you know this?
1
Not at all
2
3
4
5
Perfectly
19
Q

Which of the following are benefits typically covered under Part A of Medicare?

A

hospital nursing services

How well did you know this?
1
Not at all
2
3
4
5
Perfectly
20
Q

Which of the following statements concerning the “retirement ladder” model of retirement savings is correct?
(A) Personal savings has replaced Social Security benefits in the retirement ladder.
(B) Home equity is a possible source of funding for retirement that was not included on the old three-legged stool.
(C) Inheritances are a key retirement source because about 80 percent of baby boomers will be receiving significant inheritances.
(D) Wages are not included as a source of retirement funding because an individual cannot be retired and still work.

A

The answer is (B). Home equity was not recognized in the “three-legged stool” model of sources of retirement income, but it has been added to the expanded list included in the retirement ladder. (A) is incorrect because the retirement ladder still considers Social Security, personal savings, and employer pensions as key sources of retirement funding. (C) is incorrect because only about 15 percent of baby boomers expect significant inheritances. (D) is incorrect because many individuals who consider themselves retired will choose to do some form of work. (Chapter 20)

How well did you know this?
1
Not at all
2
3
4
5
Perfectly
21
Q

Retirement Ladder

A
Financial Independence in Retirement!
Other forms of support
Inheritances
Retiree health, long-term care, and other insurance solutions
Personal savings
Informed planning
Fiscal welfare/social assistance
Part-time wages
Company-sponsored retirement plans
Social Security
Home equity
life insurance
Rental income
How well did you know this?
1
Not at all
2
3
4
5
Perfectly
22
Q

Which of the following documents must be provided automatically to all participants in a qualified plan?

A

summary plan description

The annual report is filed with the government and is not furnished to participants, although they may request to see it. Participants have a right to see and copy the plan document and trust agreement. However, it need not be provided automatically to them.

How well did you know this?
1
Not at all
2
3
4
5
Perfectly
23
Q

Which of the following statements concerning the fee disclosure rules that apply to service providers is correct?

(A) The rules apply to SEPs and SIMPLEs.
(B) The rules only require that service providers report direct compensation.
(C) If the rules are not followed the service providers are charged a $1,000 fine.
(D) The rules apply to those providing brokerage services to a 401(k) plan with participant-directed accounts.

A

The answer is (D). (A) is incorrect because the fee disclosure rules do not apply to SEPs and SIMPLEs. (B) is incorrect because covered service providers must disclose both direct and indirect compensation. (C) is incorrect because if the rules are not followed the fiduciary has engaged in a prohibited transaction and the service provider can be subject to a penalty tax, which is 15 percent of the amount involved. (Chapter 12)

How well did you know this?
1
Not at all
2
3
4
5
Perfectly
24
Q

Which of the following statements concerning the minimum funding requirements is correct?
(A) Plans that are “at risk” are given additional time to fund the plan.
(B) Fully insured (Code Sec. 412(e)) plans are subject to additional minimum funding requirements.
(C) The funding target normal cost is the cost for the current year’s benefit accruals.
(D) The rules allow a plan sponsor to choose from a wide range of actuarial cost methods.

A

The answer is (C). (A) is incorrect because at-risk plans are subject to more accelerated funding. (B) is incorrect because fully insured plans are exempt from the minimum funding requirements (D) is incorrect because the rules today require all sponsors to use the same method for determining costs; in the past employer’s had more choice. (Chapter 11)

How well did you know this?
1
Not at all
2
3
4
5
Perfectly
25
Q

Which of the following statements concerning money-purchase pension plans is correct?
(A) They must contain a specified contribution formula.
(B) They are covered by the PBGC.
(C) They must contain a formula for allocating profits.
(D) They are difficult to administer.

A

The answer is (A). Because contributions are tied to each year’s compensation, the benefit grows relatively evenly over the accumulation period. This is good for employees who change jobs often, but may not be good for a long-term employee in an economic environment of high inflation in the years just preceding retirement. In this environment, a defined-benefit plan that bases benefits on final-average compensation can better protect the participant. (B) is incorrect because it refers to defined-benefit plans. (C) is incorrect because it refers to profit-sharing plans. (D) is incorrect because these plans are relatively easy to administer. (Chapter 4)

How well did you know this?
1
Not at all
2
3
4
5
Perfectly
26
Q

Which of the following statements correctly describes a simplified employee pension (SEP) plan?

(A) The plan can exclude employees who are aged 25 or younger.
(B) The plan must provide for immediate and full vesting.
(C) The plan can exclude employees who work fewer than 1,000 hours a year.
(D) The allocation formula cannot be integrated with Social Security.

A

The answer is (B). (A) is incorrect because the plan can only exclude employees from participation prior to age 21. (C) is incorrect because the plan can disregard only those part-time employees who earn less than a specified dollar limit ($600 in 2017). (D) is incorrect because SEPs can have an allocation formula that is integrated with Social Security. (Chapter 6)

How well did you know this?
1
Not at all
2
3
4
5
Perfectly
27
Q

Which of the following statements concerning the design of a 401(k) plan is (are) correct?
I. All types of plans have minimum vesting requirements.
II. The employer always receives a deduction at the time contributions are made.

A

Both I and II

How well did you know this?
1
Not at all
2
3
4
5
Perfectly
28
Q

Which of the following statements concerning profit-sharing plans is (are) correct?
I. The employer can choose from a number of different allocation formulas.
II. Profit-sharing plans are popular because contributions may be made on a discretionary basis.

A

Both I and II

How well did you know this?
1
Not at all
2
3
4
5
Perfectly
29
Q

Which of the following statements concerning the cost of living in retirement is (are) correct?
I. Income taxes are reduced because a nonworking person is not subject to FICA taxes and the individual is more likely to be eligible for a deduction for heath care expenses.
II. One of the largest expenses that goes away in retirement is the expense of saving for retirement.

A

Both I and II

How well did you know this?
1
Not at all
2
3
4
5
Perfectly
30
Q

Which of the following statements concerning retirement security concerns for women is (are) correct?
I. Women have less income for retirement because they tend to work in jobs that do not provide pension benefits.
II. Women may have less income for retirement because they are more likely to leave a job to become caregivers then men.

A

Both I and II

How well did you know this?
1
Not at all
2
3
4
5
Perfectly
31
Q

Which of the following statements concerning the expense method used in retirement planning is (are) correct?
I. The expense method of retirement planning focuses on the percentage of a person’s final salary that should be provided for retirement.
II. The expense method of retirement planning is more precise for younger than for older people.

A

Neither I nor II
I is incorrect because the expense method focuses on the projected expenses a retiree will have in the first year of retirement. II is incorrect because the expense method is more precise for older people than for younger people. (Chapter 21)

How well did you know this?
1
Not at all
2
3
4
5
Perfectly
32
Q

Which of the following statements concerning rules that apply to 401(k) plans is (are) correct?
I. The ADP test can have the result of limiting the salary deferral contributions of the highly compensated employees.
II. Employee salary deferral contributions can only be distributed in-service prior to age 59½ if the employee has an unforeseen emergency.

A

I only

II is incorrect because the standard is financial hardship, which under the safe-harbor rules includes purchasing a primary residence or paying for a child’s college education expenses. The unforeseen emergency standard is more difficult to satisfy, it is a rule that applies to nonqualified plans under Code Sec. 409A. (Chapter 5)

How well did you know this?
1
Not at all
2
3
4
5
Perfectly
33
Q

Which of the following statements concerning post-ERISA legislative changes that have an effect on retirement planning is (are) correct?
I. The laws have been changed so that today corporations and self-employed individuals have virtually the same opportunities (with a few minor exceptions) under the pension rules.
II. The laws have added special tax advantages for distributions from qualified plans.

A

I only

II is incorrect because, over the years, some of the special tax rules that apply to qualified plan distributions have been repealed. (Chapter 2)

How well did you know this?
1
Not at all
2
3
4
5
Perfectly
34
Q

Which of the following statements about 403b. (B) plans is (are) correct?
I. A 403b. (B) plan can be designed to include independent contractors.
II. A 403b. (B) plan can allow participants to make Roth elections on their salary deferral contributions.

A

II only

I is incorrect because 403(b) plans are not allowed to cover independent contractors. (Chapter 6)

How well did you know this?
1
Not at all
2
3
4
5
Perfectly
35
Q

Which of the following statements concerning the coverage requirements that apply to qualified plans is (are) correct?
I. The 401a. (A)(26) minimum-participation rule applies only to defined-benefit plans.
II. If an employer has separate lines of business, the 410b. (B) tests may be applied separately in each line of business when the businesses are operated for bona fide business reasons, have at least 50 employees, and meet several other regulatory requirements.

A

Both I and II

How well did you know this?
1
Not at all
2
3
4
5
Perfectly
36
Q

Which of the following statements concerning the vesting break-in-service rules is (are) correct?
I. A break in service is a year in which the individual does not complete more than 500 hours of service.
II. In a defined-contribution plan, if an individual has five consecutive breaks in service, the nonvested portion of the benefit earned prior to the break can be permanently forfeited.

A

Both I and II

How well did you know this?
1
Not at all
2
3
4
5
Perfectly
37
Q

Which of the following statements concerning information sources for a pension practitioner is (are) correct?
I. Online services provide up-to-date information about laws, regulations, and related areas.
II. Primary sources include books and periodicals that provide in-depth overviews of pension plans.

A

I only

II is incorrect because primary sources are laws and regulations.

How well did you know this?
1
Not at all
2
3
4
5
Perfectly
38
Q

Which of the following statements concerning 401(k) plans is (are) correct?
I. 401(k) plan can allow for employee salary deferral contributions, employer-matching contributions, and employer profit-sharing contributions.
II. Employee salary deferral contributions can be distributed 2 years after they have been made.

A

I only

II is incorrect because employee salary deferral contributions cannot be distributed in service until the participant attains age 59½ or suffers a financial hardship. (Chapter 5)

How well did you know this?
1
Not at all
2
3
4
5
Perfectly
39
Q

Which of the following statements concerning incentive stock options (ISO) is (are) correct?
I. An ISO’s exercise price must be equal to or greater than 100 percent of the underlying stock’s fair market value on the date of grant.
II. If an employee does not satisfy the ISO’s holding period requirements, the gain to the extent of the spread at the time of exercise is taxed as ordinary income.

A

Both I and II

How well did you know this?
1
Not at all
2
3
4
5
Perfectly
40
Q

Which of the following statements concerning 401(k) plans with safe harbor contributions is (are) correct?
I. The safe harbor option that is available for plans with automatic enrollment may require two years of service before participants are fully vested in the safe harbor contribution.
II. A contribution of 3 percent of compensation for all nonhighly compensated employees eligible to participate in the plan satisfies the safe harbor contribution requirements.

A

Both I and II

How well did you know this?
1
Not at all
2
3
4
5
Perfectly
41
Q

Which of the following statements concerning a rabbi trust is (are) correct?
I. It can have an “insolvency trigger” in which benefits are paid to the executive if the company begins to fail financially.
II. It is a way to protect participants from a corporate takeover without triggering taxation at the time contributions are made to the trust.

A

II only
I is incorrect because a rabbi trust cannot contain an insolvency trigger. This would be a way to circumvent the requirement that assets in the trust must be available to the company’s creditors. (Chapter 15)

How well did you know this?
1
Not at all
2
3
4
5
Perfectly
42
Q

What is a Rabbi Trust and what are the primary benefits?

Assets must remain available for who?

A

With the rabbi trust (first conceived in 1981 to provide benefit security for a rabbi),
contributions are made to a separate trust. Under the terms of the trust, assets generally cannot
revert to the company—meaning that plan assets will be available to pay plan benefits, even
if new hostile management takes over the company. However, to avoid current taxation to the
participants, the trust’s assets remain subject to the claims of the employer’s creditors.

How well did you know this?
1
Not at all
2
3
4
5
Perfectly
43
Q

Which of the following statements concerning cross-tested defined-contribution plans is (are) correct?
I. In a cross-tested plan, contributions allocated to participants are tested for nondiscrimination by first converting the contributions to equivalent benefit accruals.
II. An age-weighted plan is a type of cross-tested plan that allocates the amount necessary so all participants receive the same allocation of contributed funds (as a percentage of salary).

A

I only

Statement II is describing a level allocation of salary formula. (Chapter 8)

How well did you know this?
1
Not at all
2
3
4
5
Perfectly
44
Q

Longest possible vesting period?

A

7 years

How well did you know this?
1
Not at all
2
3
4
5
Perfectly
45
Q

Which of the following statements concerning pension portfolio investments is (are) correct?
I. Bond investments can be used to create sufficient cash flow to satisfy benefit payouts.
II. A plan purchasing an operating car wash could result in the assessment of the unrelated business income tax.

A

Both I and II

How well did you know this?
1
Not at all
2
3
4
5
Perfectly
46
Q

Which of the following statements concerning disability benefits in a qualified plan is (are) correct?
I. An employer is required to fully vest participants when they become disabled.
II. Many plans give credit for those on disability for years of benefit service.

A

Neither I nor II
I is incorrect because full vesting at disability is common but not required. II is incorrect because providing benefit service to disabled participants can be costly, and most employers provide disability benefits outside of the qualified plan environment. (Chapter 10)

How well did you know this?
1
Not at all
2
3
4
5
Perfectly
47
Q

Which of the following statements concerning the prohibited transaction rules is (are) correct?
I. A sale of an investment to the plan from a party in interest is a prohibited transaction unless it is exempted by a statutory, administrative, or individual exemption.
II. The plan’s attorney is not a party in interest.

A

I only
II is incorrect because all service providers to the plan, including an attorney, are considered parties in interest. (Chapter 12)

How well did you know this?
1
Not at all
2
3
4
5
Perfectly
48
Q

Which of the following statements concerning IRA rollovers is (are) correct?
I. An IRA account can never be rolled into a 403b. (B) plan.
II. A qualified plan benefit can generally be rolled into an IRA.

A

II only
I is incorrect because an IRA can be rolled into or transferred into another IRA, qualified plan, 403(b) plan, and a even government sponsored 457 plan. (Chapter 24)

How well did you know this?
1
Not at all
2
3
4
5
Perfectly
49
Q

Which of the following statements concerning qualified domestic relations orders (QDROs) is (are) correct?
I. The divorce court determines whether a domestic relations order is qualified or not.
II. Only a spouse or former spouse can be an alternate payee under a QDRO.

A

Neither I nor II
I is incorrect because the plan administrator must determine whether the court order meets the qualification requirements. II is incorrect because a child or other dependant can also be an alternate payee under a QDRO. (Chapter 13)

How well did you know this?
1
Not at all
2
3
4
5
Perfectly
50
Q

Characteristics of a properly structured nonqualified plan include which of the following?
I. fully secured benefit promises
II. deferral of taxation until benefit receipt

A

II only

I is incorrect because fully secured promises would be taxable as soon as benefits became fully vested. (Chapter 15)

How well did you know this?
1
Not at all
2
3
4
5
Perfectly
51
Q

Which of the following statements concerning the tax treatment of nonqualified plans is (are) correct?
I. Salary deferrals are not included in income as long as they are subject to a substantial risk of forfeiture.
II. Salary deferrals are not included in income as long as the participant does not have current possession of the benefits.

A

I only
II is incorrect because a participant could have taxable income without having possession since the economic benefit doctrine or the requirements of Code Sec. 409A are not followed. (Chapter 15)

How well did you know this?
1
Not at all
2
3
4
5
Perfectly
52
Q

Which of the following statements concerning the ERISA 404c. (C) participant-directed individual account plan exception is (are) correct?
I. The exemption means that the fiduciaries are not responsible for the investment alternatives available to plan participants.
II. The fiduciary relief can not apply to an individual who fails to make affirmative investment elections.

A

Neither I nor II
I is incorrect because the fiduciaries remain responsible for the investment alternatives that they choose to include in the plan. II is incorrect because there is an exception to the general rule that the participant must make an affirmative election if the assets are invested in a qualified default investment alternative and certain disclosure requirements are satisfied. (Chapter 12)

How well did you know this?
1
Not at all
2
3
4
5
Perfectly
53
Q

For individual retirement account (IRA) contribution purposes, which of the following statements concerning who is an active participant in an employer-maintained retirement plan is (are) correct?
I. Generally, a person is an active participant in a defined-benefit plan unless excluded under the plan’s eligibility provision.
II. Generally, a person is an active participant in any type of defined-contribution plan if employer contributions or forfeitures are allocated to the individual’s account for the year.

A

Both I and II

How well did you know this?
1
Not at all
2
3
4
5
Perfectly
54
Q

Which of the following statements concerning retirement investing during the accumulation period is (are) correct?
I. Annual retirement funding during this period should be concentrated in investments of employer stock.
II. A tax efficient way to save for retirement is to invest in tax-deferred accounts (such as IRAs in stock) and invest taxable accounts in bonds.

A

Neither I nor II
I is incorrect because clients are not well diversified if they concentrate on employer stock. II is incorrect because it is better to invest in bonds in a tax deferred account and better to invest in stock (because of its special tax treatment) in taxable accounts. (Chapter 21)

How well did you know this?
1
Not at all
2
3
4
5
Perfectly
55
Q

Which of the following statements concerning a life annuity is (are) correct?
I. If a client dies within one year of a life annuity’s starting date, the amount of the purchase price less any payments made is distributed to the client’s beneficiary.
II. All else being equal, a life annuity will provide a lower monthly benefit than a joint-and-survivor annuity.

A

Neither I nor II
I is incorrect because there are no death benefits paid out under a life annuity. II is incorrect because, all other things being equal, a life annuity will provide a higher monthly benefit than a joint-and-survivor annuity. (Chapter 25)

How well did you know this?
1
Not at all
2
3
4
5
Perfectly
56
Q

Which of the following statements concerning retirement planning for individuals is (are) correct?
I. A worker’s income can fall by the amount being saved for retirement with no concurrent reduction in standard of living.
II. A replacement ratio approach is appropriate for determining the retirement needs of a young person who cannot anticipate retirement expenses.

A

Both I and II

How well did you know this?
1
Not at all
2
3
4
5
Perfectly
57
Q

Which of the following statements concerning plan distributions is (are) correct?
I. Minimum distributions from an IRA must begin as of the April 1 following the year in which the person reaches age 70½.
II. The net unrealized appreciation rule is available to a participant who receives a qualifying lump-sum distribution from a qualified plan that includes a distribution of stock of the sponsoring employer.

A

Both I and II

How well did you know this?
1
Not at all
2
3
4
5
Perfectly
58
Q

All the following statements concerning the use of an early retirement age are correct EXCEPT
(A) A plan is required to provide for early retirement benefits.
(B) A subsidized early retirement benefit provision is more costly than a provision that is not subsidized.
(C) A subsidized early retirement provision is a good idea when the employer wants to encourage older employees to retire.
(D) Common early retirement provisions allow for retirement at age 55, 60, or 62.

A

The answer is (A). Every plan must have a normal retirement provision, but early retirement is optional. (Chapter 9)

How well did you know this?
1
Not at all
2
3
4
5
Perfectly
59
Q

Assuming that the plan satisfied the Code Sec. 410b. (B) minimum coverage rule, all the following could be permitted eligibility requirements for a qualified pension plan EXCEPT
(A) Only employees of the employer’s California plant are eligible to participate.
(B) All hourly paid employees are eligible who are not members of a collective-bargaining unit.
(C) All employees who have attained age 18 are eligible to participate.
(D) All employees who have earned 4 years of service are eligible to participate.

A

The answer is (D). Even if a plan satisfied Code Sec. 410(b) the eligibility provisions relating to age and years of service are specifically restricted. The Code requires that the age requirement for eligibility in a qualified plan be no older than 21 and the required years of service for eligibility be no more than one unless immediate 100 percent vesting is provided, in which case, up to 2 years may be required. (Chapter 7)

How well did you know this?
1
Not at all
2
3
4
5
Perfectly
60
Q

Reasons why an employer should adopt a defined-benefit plan to account for past service include all the following EXCEPT

(A) Long-service employees will benefit if past service is accounted for.
(B) A past-service provision can maximize the tax-shelter potential of the plan for owner-employees.
(C) A past-service provision will lower the employer’s costs.
(D) The minimum funding requirements apply when past service is accounted for.

A

The answer is (C). A past-service provision will increase an employer’s costs. (Chapter 4)

How well did you know this?
1
Not at all
2
3
4
5
Perfectly
61
Q

All the following employees are considered highly compensated employees in 2017 EXCEPT
(A) Jane Johnson, who earns $100,000 in 2016 and is the second highest paid employee in a 25-person firm
(B) Bill James, a 10 percent owner in 2016 and 2017
(C) Cathy Carr, a 6 percent owner who earns $155,000 in 2016
(D) John Smith, an officer earning $140,000 in 2016, whose compensation is in the top 20 percent of all employees

A

The answer is (A). Jane Johnson is not highly compensated in 2017 because she neither earns $120,000 in 2016 nor is a 5 percent owner. Bill James is a highly compensated employee because he is a more-than-5-percent owner. Cathy Carr also is considered a highly compensated employee because she is a 5 percent owner. John Smith earns more than $120,000 in 2016 and is an employee whose earnings put him in the top 20 percent group. (Chapter 5)

How well did you know this?
1
Not at all
2
3
4
5
Perfectly
62
Q

All the following statements concerning profit-sharing plans are correct EXCEPT
(A) They allow for a deductible contribution of up to 25 percent of the aggregate compensation of all eligible participants.
(B) They can be designed to allow participants withdrawal flexibility.
(C) They can only be funded if the employer has profits.
(D) They can be designed to enable the employer to skip contributions as long as contributions are made on a substantial and recurring basis.

A

The answer is (C). Since 1987, contributions are allowed in profit-sharing plans even if the company has no profits. (Chapter 3)

How well did you know this?
1
Not at all
2
3
4
5
Perfectly
63
Q

All the following statements concerning nonqualified plan design features are correct EXCEPT
(A) Nonqualified plans always are structured as defined-contribution plans.
(B) Nonqualified plan participation typically is restricted to a select group of executives and highly compensated employees.
(C) Nonqualified plans can contain death benefits covering the preretirement period, the postretirement period, or both.
(D) Nonqualified plans can contain provisions for the forfeiture of benefits.

A

The answer is (A). Nonqualified plans can be structured in the defined-benefit format. (Chapter 15)

How well did you know this?
1
Not at all
2
3
4
5
Perfectly
64
Q

All the following statements concerning a defined-benefit plan are correct EXCEPT

(A) The employer typically pays the entire cost.
(B) The employer carries the risk of investment performance.
(C) The plan must base benefits on an average of 35 years of compensation.
(D) The plan can be difficult to communicate to employees.

A

The answer is (C). Defined-benefit plans could be designed this way, but it is much more likely that the benefit formula will replace final average compensation, which will be the highest 3-5 years. (Chapter 3)

65
Q

All the following statements concerning the actual deferral percentage (ADP) test for nondiscrimination in a 401(k) plan are correct EXCEPT
(A) If a plan fails the ADP test, salary deferrals of the nonhighly compensated may have to be returned to the employees.
(B) The plan stands a better chance of passing the ADP test if matching employer contributions are offered.
(C) Salary deferrals that are treated as Roth contributions are still subject to the ADP test.
(D) The plan does not have to satisfy the ADP test if a qualifying safe harbor contribution is made.

A

The answer is (A). Failing the ADP test means that the contributions for the highly compensated employees (not the nonhighly compensated) may have to be returned or recharacterized. (Chapter 5)

66
Q

All the following are correct regarding phased retirement programs in the United States EXCEPT

(A) most phased retirement programs are informal programs
(B) phased retirement programs can help a retiree find a better work life balance and test drive retirement before fully committing to a new stage of life in retirement.
(C) most retirees do not use a phased retirement program before retiring
(D) large employers with over 200 employees are required under the ADEA (age discrimination act) to offer a phased retirement program for employees over 70.

A

The answer is (D). There is no federal requirement to offer a phased retirement program, it is entirely optional for employers. (Chapter 22)

67
Q

Provisional Income

A

If a married taxpayer has provisional income in excess of $32,000, some of the Social Security will be taxed. income includes adjusted gross income, as well as the taxpayer’s tax-exempt interest and one-half of Social Security benefits.

68
Q

All the following statements concerning retirement ages in a defined-benefit plan are correct EXCEPT
(A) The normal retirement age is the age specified in a plan at which a participant can retire with full benefits.
(B) Deferred-retirement provisions must provide for additional benefit accruals unless the benefit formula has a service cap.
(C) The oldest age that can be set for the normal retirement age is the later of age 70 or 10 years of participation.
(D) An early retirement age is optional.

A

The answer is (C). The oldest age that can be set for the normal retirement age is the later of age 65 or 5 years of participation. (Chapter 9)

69
Q

…….. has a specified benefit formula and the actuary then determines the required contribution to fund the targeted benefit.

A

Defined Benefit Plan

70
Q

……… plan identifies a targeted benefit at retirement, and contributions are made in the amount
necessary to fund the targeted amount. The plan specifies the actuarial method and interest
rates used to determine annual contributions so that the amount of contribution can be clearly
determined. Unlike a defined-benefit plan, however, the contribution amounts will not change
each year based on the value of the plan’s assets. Contributions only change to reflect new
plan participants and increases in the compensation of existing plan participants.

A

Target benefit Plan

71
Q

……. plan contributions change year by year based on the plan asset value.

A

Defined Benefit Plan

72
Q

………. formulas provide the same benefit for all participants.

A

Flat amount formulas

73
Q

……… formulas with service caps can be used to help turn over the work force.

A

Unit Benefit
The most frequently used defined-benefit formula is the unit-benefit formula (also known as the percentage-of-earnings-per-year-of-service formula). This formula uses both service and salary in determining the participant’s pension benefit. A unit-benefit formula might read this
way: “Each plan participant will receive a monthly pension commencing at normal retirement date and paid in the form of a life annuity equal to 1.5 percent of final-average monthly salary multiplied by years of service. Service is limited to a maximum of 30 years.”

74
Q

Advisory opinions-communications similar to private-letter rulings-are distributed by the

A

Department of Labor.

75
Q

private-letter rulings, proposed regulations and revenue rulings are all communications issued by:

A

The IRS

76
Q

All the following statements concerning reverse mortgages are correct EXCEPT
(A) They are typically available only when the owners are aged 62 or older and when the home is the principal residence.
(B) The amount of loan payments made depends on the owner’s age, the amount of equity in the home, and the interest rate and fees that are being charged.
(C) The federally sponsored Home Equity Conversion Mortgage (HECM) program is the most popular type of reverse mortgage.
(D) They typically have to be repaid after a specified number of years.

A

The answer is (D). Typically, a reverse mortgage only has to be repaid when the last surviving borrower dies, sells the home, or moves away permanently. (Chapter 23)

77
Q

The total Social Security tax rate on self-employment income is higher than that paid on the covered income of persons employed by others.

A

False. The tax rate for both employed and self-employed persons is 15.30 percent. However, for employed persons, the tax rate is divided equally between employee and employer.

78
Q

The Medicare tax for an employee is 1.45 percent on all income from employment.

A

True

79
Q

After full retirement age, once a person begins receiving Social Security benefits, he or she no longer has to pay FICA tax on any earned income.

A

False. Earned income is subject to FICA tax even if it is earned after normal retirement age and after commencement of retirement benefits.

80
Q

If a married person is currently insured when he or she dies, survivor benefits are payable to the spouse at 100 percent of the deceased person’s PIA if the surviving spouse begins receiving benefits at the full retirement age.

A

False. A surviving spouse will receive survivor benefits equal to 100 percent of the deceased spouse’s PIA at the full retirement age only if the deceased spouse was fully insured. A person is fully insured only if he or she has the required number of quarters of coverage (40). A person who does not have the requisite number of quarters of coverage to qualify as fully insured will be currently insured if he or she has at least six quarters of coverage. However, if the deceased partner was only currently insured, a spouse will receive survivor benefits only if he or she is caring for an eligible child who is under age 16 or who was disabled before age 22.

81
Q

The number of years of salary history used to determine the AIME is 10 less than the number of quarters of coverage needed to be fully insured.
average indexed monthly earnings (AIME)

A

False. The number of years of salary history used to calculate the AIME is 5 less than the number of quarters of coverage needed to be fully insured.

82
Q

The total retirement benefit paid to a family based on one worker’s wages may be limited whenever there are two or more individuals receiving a benefit.

A

False. The family maximum is not a concern until there are three or more family members receiving retirement benefits based on one worker’s wages.

83
Q

The spouse of a retired worker who is receiving retirement benefits may begin receiving spousal benefits before full retirement age, but the benefit that would otherwise be received at full retirement age is reduced by 5/9 of one percent for each month before full retirement age benefits begin.

A

False. The spouse of a retired worker who is receiving retirement benefits may begin receiving spousal retirement benefits any time after he or she reaches age 62. However, the benefit is reduced by 25/36 of one percent for each month before full retirement age.

84
Q

The benefits of a person who continues to work after beginning retirement benefits at age 62 will be reduced by $….. for every $…… of earnings in excess of the earnings limitation.

A

by $1 for every $2 of earnings.

The earnings limitation is $16,920 (2017), $17,640 (2019)

85
Q

A 62-year-old who has begun Social Security benefits because of a layoff can later suspend benefits, either because of the earnings test or voluntarily after attaining full retirement age.

A

True

86
Q

Describe a sale-leaseback arrangement

A

Under a sale-leaseback arrangement, your client sells his or her house to an investor and then rents it back from the investor under a lifetime lease.

87
Q

What is sequence of returns risk?

A

Sequence of returns risk is the risk of receiving low or negative investment returns early in retirement when withdrawals are being made from those investments. Bad early years of returns in a portfolio subject to withdrawals can dramatically reduce the
longevity and sustainability of that portfolio. This means a few bad years of returns early in retirement can make a retiree run out of money later in retirement much
earlier than someone who has good investments in the first few years of retirement.

88
Q

Whats a self-directed IRA

A

Self-directed IRAs (IRAs in which the taxpayer is able to shift investments between general investment vehicles offered by the trustee) are also popular because they give the investor investment flexibility and the ability to anticipate or react to interest-rate directions and market trends.

89
Q

Deemed IRA’s

A

For financial services professionals who work primarily with employers, note that qualified plans, 403(b) annuities, and even government 457 plans may be written to allow participants to make voluntary IRA or Roth IRA contributions directly to the plan. These accounts are referred to as deemed IRAs. If a plan so provides, the document must require separate accounting for the IRA or Roth IRA contributions, and the contributions would have to meet the requirements
applicable to either traditional IRAs or Roth IRAs.

90
Q

What roles result in an individual becoming a fiduciary to a retirement plan?

A

An individual who exercises discretionary authority over the management of the plan or disposition of the plan’s assets, provides investment advice for a fee, or
exercises discretionary control over the plan’s administration will all be considered plan fiduciaries. Note that this is functional definition, and does not require that the individual be specifically named as a fiduciary.

91
Q

Name the affirmative duties that fiduciaries are required to satisfy.

A

The affirmative fiduciary obligations that apply to all types of plans are (1) to maintain the plan for the exclusive benefit of the participants, (2) to discharge fiduciary duties with the prudence of a knowledgeable investment professional, (3) to diversify plan assets, and (4) to invest plan assets in accordance with the plan’s documents. In addition, individual account plans that allow participant investment direction must satisfy the participant fee disclosure requirements.

92
Q

When can a fiduciary be held liable for a breach of duty by a cofiduciary?

A

A fiduciary can also be liable for the breach of a cofiduciary if the fiduciary actively participates in the behavior, attempts to conceal the cofiduciaries breach, or even just has knowledge of the breach and fails to take steps to remedy the situation.

93
Q

How can an employer protect employees who act as plan fiduciaries?

A

An employer can either purchase liability insurance for the employees or indemnify them from losses in their role of fiduciary.

94
Q

What type of plans can be integrated with social security

A

money-purchase and profit-sharing plans. Can only integrate one type of plan with social security.

95
Q

Maximum integration with social security

A

If the integration level is set at the current taxable wage base ($127,200 for 2017), HCEs may receive up to 5.7 percent of compensation in excess of the taxable wage base, as long as the employer makes contributions that equal at least 5.7 percent of total compensation.

96
Q

Which of the following statements concerning the diversification requirement that applies to defined-contribution plans that invest in securities of the employer is correct?

(A) The rule requires that eligible participants can choose from three other investment alternatives.
(B) The rule applies both to plans of publicly traded and closely held companies.
(C) The rule only applies to participants who have attained age 55.
(D) The rule applies to ESOPs.

A

The answer is (A). (B) is incorrect because the rule only applies to the stock of publicly traded companies. (C) is incorrect because the rule does not have an age restriction. (D) is incorrect because the rule does not apply to ESOPs. ESOPs are subject to a more restricted diversification requirement. (Chapter 11)

97
Q

Which of the following statements concerning the safe savings rate is correct?

(A) By waiting to a later age to start saving for retirement, an individual will lower his or her safe savings rate.
(B) By taking on more investment risk (ie. higher allocation in equities) at an early age, you can lower your safe savings rate.
(C) The longer the expected retirement period, the lower the safe savings rate will be.
(D) Safe savings rates are set for all individuals of a given age and do not depend on how much they have been saving or how their investments are allocated.

A

The answer is (B). A is incorrect because if you wait to start saving until later in life, you will have to save more to offset the lack of early savings. C is incorrect because if you need to fund a longer retirement period, you need to save more money throughout your working years. D is incorrect because the safe savings rate depends on how long you will save, your investments, and the length of your projected retirement period. (Chapter 22)

98
Q

What is human capital?

A

Human capital is an individual’s earning potential, which is
the single most valuable asset that most people have before retirement. Human capital can be expressed as the present value of wages that someone can earn over the course of their lifetime.

99
Q

Safe Savings Rate

A

The safe savings rate is the maximum of all the minimum necessary savings rates from overlapping historical periods needed to accumulate an individual’s desired wealth in order to meet a certain level of spending in retirement. The safe savings rate will tell you how much
you need to save over a 20, 30, or 40-year working career, as a percent of your salary, to replace either 50% or 70% of your final salary in retirement based on your investment asset allocation.

100
Q

Which of the following statements concerning the characteristics of nonqualified retirement plans is correct?
A) In order for the owners and managers to participate, the plan must also cover a significant number of rank-and-file employees.
(B) Although benefit amounts can discriminate in favor of highly compensated employees, vesting must be immediate.
(C) A plan can generally be designed to defer the payment of income taxes by the employee until benefits are paid out.
(D) Contributions must be made to a trust, an insurance product, or an IRA account so the plan assets are not owned by the employer.

A

The answer is (C). (A) is incorrect because nonqualified plans are generally for only a few key people. (B) is incorrect because there are few design restrictions regarding benefit structure, vesting requirements, or coverage with nonqualified plans. (D) is incorrect because, in most cases, nonqualified plans do not have separate assets. The employer either pays benefits out of general corporate assets or sets up a side account. Even if a trust is set up, the assets must be available to pay the claims of creditors in order to avoid current taxation. (Chapter 15)

101
Q
Which of the following types of retirement plans is allowed to borrow in order to purchase company stock?
(A) stock bonus plan
(B) profit-sharing plan
(C) ESOP
(D) 401(k) plan
A

The answer is (C). (A), (B), and (D) are incorrect because they do not allow the plan to borrow in order to purchase company stock. (Chapter 5)

102
Q

Which of the following statements concerning the suitability of investments for defined-benefit plans is correct?
(A) Investments that have special tax advantages, such as tax-free municipal bonds, are generally the most appropriate type of investment for qualified plans.
(B) Liquidity is an important concern when selecting plan investments because without sufficient liquidity in a portion of a plan’s assets, long-term investments might have to be sold by the plan to make benefit payments.
(C) Life insurance is generally a prudent choice for funding a qualified plan in a large corporation that also provides group life insurance for its employees.
(D) Because of the lack of stability in value, defined-benefit plans tend to invest only a very small percentage of assets in equities.

A

The answer is (B). (A) is incorrect because qualified plans already have special tax advantages and, thus, purchasing tax-advantaged investments for which the investor pays a premium for the tax advantage is generally not appropriate for the plan. (C) is incorrect because large corporations generally provide for the life insurance needs of their employees through a group life plan. Because a life insurance need must exist before funding a qualified plan with life insurance becomes prudent, large corporations seldom use life insurance to fund retirement plans. (D) is incorrect because, while stability is a consideration in defined-benefit plans, because of the long-term nature of retirement plan investments and the concern to keep long-term costs down, about half of pension plan assets are invested in equities. (Chapter 11)

103
Q

Which of the following statements concerning the taxation of Social Security benefits is correct?
(A) The maximum amount of Social Security benefits subject to tax is 50 percent.
(B) Provisional income is the amount of income that is subtracted from Social Security benefits to determine the amount of the benefits that are subject to tax.
(C) The taxpayer’s tax-exempt interest for the year is considered when determining the portion of the Social Security benefit that is subject to tax.
(D) Today, Social Security benefits received by all retirees are subject to some federal income taxation.

A

The answer is (C). (A) is incorrect because the maximum amount of Social Security benefits subject to tax is 85 percent for certain taxpayers. (B) is incorrect because provisional income is the sum of the taxpayer’s adjusted gross income, the taxpayer’s tax-exempt interest for the year, and half of the Social Security benefits for the year. The provisional income is used in determining the amount of the taxpayer’s Social Security benefits that are subject to federal income tax. (D) is incorrect because Social Security benefits received by retirees whose provisional income is less than the base amount are not taxable. (Chapter 19)

104
Q
What annual form must be filed for the year if a sole proprietor of an unincorporated business maintains a profit-sharing plan that only covers the sole proprietor with $295,000 of assets at the beginning of the year?
(A) Form 5500
(B) Form 5500-EZ
(C) No annual report required
(D) Form 5300
A

The answer is (B). A “one person plan” with more than $250,000 in assets is required to file Form 5500-EZ. If assets are less than $250,000, then no filing is required. (Chapter 13)

105
Q

Which of the following statements concerning defined-benefit plans is correct?
(A) They have a predetermined cost to the employer.
(B) If the benefit is tied to final average salary they can provide employees with preretirement inflation protection.
(C) The risk of the plan’s investment experience falls primarily on the employee.
(D) They are generally less costly to administer than defined-contribution plans.

A

The answer is (B). (A) is incorrect because defined-benefit plans provide a fixed predetermined benefit that has an uncertain cost to the employer. (C) is incorrect because with defined-benefit plans, risk of the plan’s investment experience falls on the employer. (D) is incorrect because defined-benefit plans are more costly to administer than defined-contribution plans. (Chapter 3)

106
Q

Which of the following statements concerning the actual deferral percentage (ADP) nondiscrimination test for a 401(k) plan is correct?
(A) Under current-year testing, the ADP of the highly compensated group is limited by the ADP of the nonhighly compensated group in the previous year.
(B) Salary deferrals designated as Roth 401(k) contributions are not subject to the ADP test.
(C) If the employer makes a safe harbor contribution, the ADP test does not have to be performed.
(D) The plan automatically becomes disqualified if it does not satisfy the ADP test at the end of the testing year.

A

The answer is (C). (A) is incorrect because, under current-year testing, the ADP limit for the HCE group is tied to the deferral percentage of the nonhighly compensated employees during the current year. (B) is incorrect because contributions designated as Roth contributions are still treated as salary deferrals for ADP testing purposes. (D) is incorrect because the administrator has the opportunity to correct the problem after the plan year ends. (Chapter 5)

107
Q

Which of the following statements concerning nonqualified stock options (NQSOs) is correct?
(A) At the time the option is exercised, the increase in the value of the stock since the option was granted is taxed as capital gain to the employee.
(B) NQSOs must be immediately 100-percent vested at the time they are granted.
(C) The option price normally equals 100 percent of the stock’s fair market value on the date the option is granted, but it may be set below this level.
(D) At the time a NQSO is granted, it is subject to income tax withholding.

A

The answer is (C). (A) is incorrect because, at the time the option is exercised, the increase in the stock’s value since the option was granted is taxed as ordinary income to the employee. (B) is incorrect because many NQSOs are not vested (and, thus, not exercisable) for a period of time after they are granted. A number of different vesting approaches are used. (D) is incorrect because, at the time an NQSO is exercised, it is subject to income tax withholding. (Chapter 16)

108
Q
In 2017 Ralph, a 44-year-old married taxpayer filing jointly, is an active participant in an employer-sponsored retirement plan. If Ralph and his spouse’s AGI is $104,000 what is the maximum deductible contribution for Ralph to a traditional IRA for 2017?
(A) $6,500
(B) $5,500
(C) $4,125
(D) $1,750
A

The answer is (C). (A), (B), and (D) are incorrect because the deductible amount of contributions is $4,125. Since Ralph is under age 50 the maximum contribution is $5,500. However because of his AGI of $104,000, $5,500 is reduced by 25 percent which is calculated by taking $104,000 – $99,000 and dividing by $20,000. (Chapter 17)

109
Q
A qualified plan provision that forces participants to take single sum distributions when the value of the benefit is less than $5,000 is called a(n)
(A) single life annuity
(B) lump sum distribution
(C) involuntary cash out
(D) installment payment
A

The answer is (C). (A), (B) and (D) are incorrect because the correct name for this plan provision is an involuntary cash out. Many qualified plans contain such a provision in order to simplify administration. (Chapter 25)

110
Q

Which of the following statements concerning SIMPLEs is correct?
(A) There is a mandatory employer contribution requirement.
(B) Elective pretax contributions by employees are not permitted.
(C) Participant loans may be permitted if in-service withdrawals are not allowed by the plan.
(D) Plan assets can be invested in life insurance.

A

The answer is (A). (B) is incorrect because elective pretax contributions by employees are a key feature of SIMPLEs. (C) is incorrect because SIMPLEs, do not permit participant loans and in-service withdrawals must be allowed by the plan. (D) is incorrect because plan assets cannot be invested in life insurance or collectibles. (Chapter 6)

111
Q

Which of the following statements is an example of a unit-benefit formula used in defined-benefit plans?
(A) a life annuity equal to 40 percent of the final-average monthly salary, commencing at normal retirement date and reduced pro rata for participants with less than 25 years of service
(B) a life annuity paid monthly, equal to $10 for every year worked, and commencing at normal retirement date
(C) a life annuity commencing at normal retirement date equal to 2 percent of final-average monthly salary, multiplied by years of service up to a maximum of 30 years
(D) a life annuity of $200 per month commencing at normal retirement date

A

The answer is (C). (A) is incorrect because it is an example of a fixed benefit (flat-percentage-of-earnings) formula. (B) is incorrect because it is an example of a flat-amount-per-year-of-service formula. (D) is incorrect because it is an example of a flat-amount formula. (Chapter 4)

112
Q

Which of the following groups is not covered under the Social Security program?
(A) all civilian employees of the federal government
(B) all employees of state and local governments
(C) all ministers who elect out of coverage because of religious principles or conscience
(D) all American citizens who work abroad for foreign affiliates of U.S. employers

A

The answer is (C). (A), (B), and (D) are incorrect because Social Security covers civilian employees of the federal government hired after 1984, employees of most state and local governments, and some American citizens who work abroad for foreign affiliates of U.S. employers. (Chapter 19)

113
Q
Suppose Janet, who has an IRA, turned 70½ on August 31, 2017, but did not retire until June 30, 2018. What is her first distribution year under the minimum-distribution rules?
(A) the year ending December 31, 2017
(B) the year ending December 31, 2018
(C) the year ending December 31, 2019
(D) the year ending December 31, 2020
A

The answer is (A). (B) is incorrect because, although the first distribution can be delayed up until the required beginning date of April 1, 2018, the distribution is for the year ending December 31, 2017. If Janet delays, she’ll have to take a minimum distribution for the second distribution year by December 31, 2018. (C) and (D) are incorrect because there are no exceptions extending the first distribution year or the required beginning date for IRAs. (Chapter 24)

114
Q

Which of the following statements concerning nonqualified salary reduction plans and supplemental executive retirement plans (SERPs) is correct?
(A) Forfeiture provisions are very common in both nonqualified salary reduction plans and SERPs.
(B) Participation in both nonqualified salary reduction plans and SERPs is typically restricted to a select group of management or highly compensated employees.
(C) Both nonqualified salary reduction plans and SERPs are always set up as defined-contribution plans.
(D) Both nonqualified salary reduction plans and SERPs provide additional benefits funded entirely by employer contributions.

A

The answer is (B). (A) is incorrect because, although forfeiture provisions are very common in SERPs, nonqualified salary reduction plans typically do not contain forfeiture provisions. (C) is incorrect because, although SERPs can be set up as defined-contribution plans, they are more frequently set up as defined-benefit plans. (D) is incorrect because, unlike SERPs, which provide additional benefits funded by employer contributions, nonqualified salary reduction plans provide a method for executives to defer current income. (Chapter 15)

115
Q

Nonqualified Plan Objectives

A
  • Alternative to qualified plan
  • Second tier of benefits
  • Cover a select group of highly paid employees
  • Salary deferral for executives
  • Instant benefit program for executives of a new company
  • Meet a wide range of compensation goals
  • Satisfy special needs of specific highly compensated employees
116
Q

What does SERP stand for

A

supplemental executive retirement plan

117
Q

Which of the following plan contributions can be subject to either 5-year cliff vesting or 3-through-7-year graded vesting?

(A) contributions to a SEP
(B) employee after-tax contributions to a 401(k) plan
(C) employer contributions to a defined-benefit plan
(D) employer contributions made to satisfy the 401(k) safe harbor provisions

A

The answer is (C). (A) is incorrect because contributions to a SEP are fully vested at all times. (B) is incorrect because, like employee pretax salary deferral elections, employee after-tax contributions to a 401(k) plan must be 100 percent vested at all times. (D) is incorrect because employer contributions made to satisfy the 401(k) safe harbor provisions must be fully vested. (Chapter 9)

118
Q
Which of the following minimum-coverage requirements imposed by the Internal Revenue Code applies only to defined-benefit plans?
(A) the average-benefit test
(B) the minimum-participation rule
(C) the ratio test
(D) the percentage test
A

The answer is (B). (A), (B), and (C) are incorrect because the percentage test, ratio test, and average-benefit test apply to all qualified plans. A plan must satisfy one of these tests to meet the coverage requirements under the Sec. 410 (b) rule. (Chapter 7)

119
Q

average-benefit percentage test

A

Another way to satisfy the minimum-coverage requirements. The group covered represents a reasonable classification, a complex minimum percentage test is satisfied, and the benefits of nonhighly compensated employees average at least 70 percent of the benefits provided to HCEs,
looking at all of the sponsor’s retirement plans

120
Q

Percentage test

A

the plan covers at least 70 percent of nonhighly compensated employees.

121
Q

Ratio test

A

the percentage of nonhighly compensated employees covered is at least 70 percent of the percentage of HCEs covered under the plan.

122
Q

Three Minimum-Coverage Tests

A

Percentage test
Ratio test
average-benefit percentage test

123
Q

Which of the following statements concerning Social Security retirement benefits is correct?
(A) After earning 40 quarters of coverage, a person is fully insured for retirement benefits for life, even if covered employment under Social Security ceases for many years prior to retirement.
(B) When a fully insured worker reaches full retirement age, retirement benefits are automatically paid if they have not begun earlier.
(C) A fully insured worker is eligible to receive a benefit equal to his or her primary insurance amount at age 62.
(D) Under the earnings test, retirement benefits are reduced only if the individual goes back to work after attaining full retirement age.

A

The answer is (A). (B) is incorrect because workers must apply for retirement benefits. (C) is incorrect because a fully insured worker will receive a reduced benefit at age 62. (D) is incorrect because retirement benefits are not affected by an individual working after attaining full retirement age. Benefits are only reduced by wages earned prior to full retirement age. (Chapter 19)

124
Q

Which of the following statements concerning age restricted housing is correct?
(A) Age-restricted housing is restricted to mobile home parks.
(B) Age-restricted housing facilities are not allowed by law to provide common recreational facilities.
(C) Age-restricted housing can require all residents to be over age 67.
(D) Age restricted housing can require all residents to be over age 62.

A

The answer is (D). (A) is incorrect because housing options include apartment buildings, retirement hotels, condominiums, and subdivisions, as well as mobile home parks. (B) is incorrect because they often provide common recreational facilities. (C) is incorrect because the correct age is 62. (Chapter 23)

125
Q
If the PBGC initiates a plan termination on a defined-benefit plan that is severely underfunded, it is called a(n)
(A) partial termination
(B) involuntary termination
(C) distress termination
(D) standard termination
A

The answer is (B). When the PBGC initiates the termination process, it is called an involuntary termination. (Chapter 14)

126
Q
Pete is a sole proprietor (unincorporated entity) with no other employees. Before he takes a deduction for contributions to his profit-sharing plan, Pete’s net earnings are $50,000. His deduction for self-employment taxes is $3,000. What is the maximum contribution that Pete can make to his profit-sharing plan for the year?
(A) $12,500
(B) $10,000
(C) $9,400
(D) $7,500
A

The answer is (C). (A) is incorrect because the limit for a sole proprietor is not 25 percent of net earnings, it is 20 percent of net earnings reduced by the amount of the Social Security deduction. (B) is incorrect because the limit is 20 percent of earnings reduced by the amount of the Social Security deduction. (D) is incorrect because the limit is not 15 percent of net earnings. (Chapter 3)

127
Q

Which of the following statements concerning the definition of compensation allowed to be used in qualified plan benefit or contribution formulas is (are) correct?
I. The definition of compensation is limited by law to regular salary and wages and, thus, may not include overtime, bonuses, or other nonrecurring compensation.
II. Annual compensation considered in the benefit or contribution formula is limited by law to the $270,000 (2017) compensation cap.

A

II only.
I is incorrect because the definition of compensation can include or exclude overtime, bonuses, and other nonrecurring compensation. (Chapter 8)

128
Q

Under an incentive stock option plan, Sam is granted an option to purchase 100 shares of company stock at the stock’s current market price of $20 per share at any time over the next 10 years. Sam exercises the option after 5 years when the stock’s market price is $40 per share. Needing cash, Sam sells all of the stock 8 months later for $50 per share. Which of the following statements concerning the tax treatment of the stock’s increase in value is (are) correct?
I. The gain of $20 per share from the date the option was granted until the date it was exercised would be taxed as a long-term capital gain.
II. The gain of $10 per share from the date the option was exercised until the stock was sold would be taxed as ordinary income.

A

Neither I nor II
I is incorrect. Because the holding-period requirements were not met, the gain of $20 per share from the date the option was granted until the date it was exercised would be taxed as ordinary income. II is incorrect. Because the holding-period requirements were not met, the gain of $10 per share from the date the option was exercised until the stock was sold would be taxed as capital gain. (Chapter 16)

129
Q

How are Employee Stock Purchase Plans taxed (ESPP’s)

A

Typically a company gives the option to employees to purchase stock at a reduced amount for a given period of time.
At the time stock is purchased under an ESPP, there are no tax consequences. Taxes are paid only at the time of the sale of the stock, and the tax consequences depend upon whether a holding period requirement has been satisfied. The requirement is that the stock is not sold
within 2 years after the date that the option is granted , i.e., typically the first day of the offering period, nor within one year after the shares are purchased, typically the last day of the offering period. When the holding period is satisfied and there is a gain from the sale, the gain is ordinary income up to the amount by which the stock’s fair market value, when the option was granted, exceeded the option price. Any excess gain is capital gain. If there is a loss from the sale, it is a capital loss, and there is no ordinary income or loss. The employer gets no deduction when the holding period requirements have been satisfied.

130
Q

Which of the following statements concerning target-benefit pension plans is (are) correct?
I. Even though they are defined-contribution plans, they share some features of a defined-benefit plans as contributions are determined based on funding a monthly benefit at retirement.
II. The employer bears the risk of investment performance in a target-benefit pension plan.

A

I only
II is incorrect because the plan participant bears the risk of investment performance as the participant’s benefit is the accumulated account balance. (Chapter 4)

131
Q

Which of the following statements concerning Medicare is (are) correct?
I. Part A, the hospital portion of Medicare, is available at no monthly cost only to anyone receiving Social Security retirement benefits.
II. Even those who have begun Social Security benefits must apply for Medicare benefits when they are eligible.

A

Neither I nor II
I is incorrect because Part A, the hospital portion of Medicare, is available at no monthly cost to any person aged 65 or older, as long as the person is entitled to Social Security or railroad retirement benefits and regardless of whether or not he or she is actually receiving those benefits. II is incorrect because persons receiving Social Security retirement benefits are automatically enrolled in both Part A and Part B of Medicare, and must elect out in writing if they do not want Part B. (Chapter 23)

132
Q

Jane Johnson is a 70-percent owner of QRS Corporation, a small closely held business with a 401(k) plan. Which of the following plan transactions is (are) a prohibited transaction?
I. The plan purchases or leases property from Jane.
II. The plan lends money to QRS Corporation.

A

Both I and II

133
Q

Which of the following statements concerning planning under the minimum-distribution rules is (are) correct?
I. After the participant’s death, the chosen beneficiary can name a new beneficiary that was not on the original designation form in order to stretch out the required distribution payments.
II. A strategy to maximize the length of the required minimum distribution period must ensure that there is a source other than the pension asset to pay any estate taxes owed after the participant’s death.

A

II only

134
Q

Ebenezer, Inc. decided for cost reasons that it would like to limit the number of participants in its qualified plan. To accomplish this goal, Ebenezer can temporarily or permanently exclude employees from participation by using which of the following methods?
I. Require an employee to accumulate 3 years of service before becoming a participant.
II. Restrict an employee who is hired within 10 years of normal retirement age from becoming a participant.

A

Neither I nor II
The answer is (D). I is incorrect because the maximum deferral period is 1 year of service unless the plan has immediate vesting, in which case the waiting period can be 2 years. II is incorrect because there is no rule that allows the sponsor to exclude older employees. (Chapter 7)

135
Q

Under qualified retirement plans, which of the following statements concerning the qualified preretirement survivor annuity (QPSA) is (are) correct?
I. It is only required to be provided for those married participants who have been married for at least one year.
II. It is only required in a defined-contribution plan and not in a defined-benefit plan.

A

I only

136
Q

Which of the following statements concerning the scope of the ERISA fiduciary rules is (are) correct?
I. Decisions made by the employer to establish or terminate the plan are business decisions and are not fiduciary actions.
II. ERISA generally exempts plans that only cover the 100 percent owner of an entity.

A

Both I and II

137
Q

Which of the following statements about saving for retirement is (are) correct?
I. As the assumption about life expectancy increases, the amount needed to be saved to support the desired lifestyle increases.
II. As the assumption about the long-term inflation rate increases, the amount needed to be saved to support the desired lifestyle increases.

A

Both I and II

138
Q

Which of the following statements concerning long term care contracts is (are) correct?
I. For tax deductibility purposes, long-term care policies are essentially like health insurance policies under the federal income tax laws.
II. Long-term care policies are currently being sold as guaranteed renewable.

A

Both I and II

139
Q

Defined Benefit vs Defined contribution

A) Maximum annual benefit limited to lesser of 100%
of average compensation or $215,000
B) Maximum annual additions limited to the lesser of
100% of compensation or $54,000

A

A) Benefit

B) Contribution

140
Q

Defined Benefit vs Defined contribution

A) Not subject to the minimum participation rule
B) Satisfy Sec. 401(a)(26) minimum participation rule

A

A) Contribution

B) Benefit

141
Q

Defined Benefit vs Defined contribution

A) 5-year cliff or 7-year graded vesting
B) 3-year cliff or 6-year graded vesting

A

A) Benefit

B) Contribution

142
Q

Explain 5-Year Cliff Vesting

A

5-Year Cliff Vesting
Years of Service Percentage Vested
0–4 0
5 or more 100

143
Q

Explain 3-7-Year Graded Vesting

A

3-through-7-Year Graded Vesting
Years of Service Percentage Vested
0–2 0
3 20
4 40
5 60
6 80
7 or more 100

144
Q

Which of the following distributions from a Roth IRA is (are) a qualifying tax-free distribution?
I. A 62-year-old taxpayer who has maintained a Roth IRA for eight years withdraws the balance (which exceeds the amount of contributions) to pay for an extended vacation.
II. Because of a disability, a 43-year-old taxpayer who has maintained a Roth IRA for three years withdraws the balance (which exceeds the amount of contributions) to pay for medical expenses.

A

I only
The answer is (A). II is incorrect because even though disability is one of the “trigger events” the withdrawal does not satisfy the 5-year rule. (Chapter 24)

145
Q

Which of the following statements concerning a pension planning fact finder is (are) correct?
I. A fact finder can be used in lieu of an adoption agreement.
II. A fact finder can be used to help the client prioritize retirement plan objectives.

A

II only
The answer is (B). I is incorrect because an adoption agreement is part of a master and prototype plan that can also be helpful to focus the client, but it does not substitute for the fact finder. (Chapter 3)

146
Q

Which of the following statements concerning money-purchase pension plans is (are) correct?
I. Businesses with cash-flow problems are especially good candidates for a money-purchase pension plan.
II. Money-purchase pension plans have the advantages of predictable cost for the employer and understandability for the employees.

A

II only
The answer is (B). I is incorrect because the employer is required to fund a money-purchase pension plan annually, which requires a steady cash flow. (Chapter 4)

147
Q
Qualified Plan Categories
Defined-Benefit Plans
1)
2)
Defined-Contribution Plans
1) 
2)
3)
4)
5)
6)
A
Defined-Benefit Plans
1) Defined-benefit pension plan
2) Cash-balance pension plan
Defined-Contribution Plans
1) Money-purchase pension plan
2)Target-benefit pension plan
3) Profit-sharing plan
4) 401(k) plan
5) Stock bonus plan
6) ESOP
148
Q

Ron, aged 35, a single taxpayer who works for a small firm with no retirement plan, with AGI of $55,000. Which of the following statements concerning Ron’s situation is (are) correct?
I. Ron could contribute both $5,500 to a traditional IRA and $5,500 to a Roth IRA.
II. If Ron makes an excess contribution to either plan and fails to withdraw the excess amount plus interest by the tax deadline in the year the excess contribution is made, he will be subject to a 6 percent excise tax penalty on the excess.

A

II only

149
Q

Which of the following statements concerning dollar-cost averaging (DCA) is (are) correct?
I. DCA is consistent with a buy-and-hold strategy that is generally appropriate during the preretirement period.
II. A disadvantage of DCA is that, over a long period of time, the investor ends up with a higher average cost for the security than was the average acquisition price for each transaction.

A

I only
II is incorrect because an advantage of DCA is that, over a long period of time, the investor ends up with a lower average cost for the security than was the average acquisition price for each transaction. (Chapter 21)

150
Q

Which of the following statements concerning qualified domestic relations orders (QDROs) is (are) correct?
I. QDROs are issued by state courts to allow a participant’s plan assets to be used for marital property rights, or child support or alimony payments to a former spouse or dependent.
II. With a QDRO, the value of the plan assets must be reduced by a coverture fraction that automatically provides a portion of the plan assets to a spouse and each dependent child.

A

I only
II is incorrect because if participation in the plan has extended over a period longer than the marriage, the value of the plan assets must be reduced by a coverture fraction that is based on the relation between the length of the marriage and the duration of the plan coverage. (Chapter 13)

151
Q

Which of the following statements concerning regular rollovers from one IRA to another is (are) correct?
I. With a rollover, a participant has the opportunity to use the money for 60 days.
II. The entire amount that qualifies for a rollover must be rolled over or the entire distribution will be subject to income tax and the penalty tax, if applicable.

A

I only

II is incorrect because the entire amount qualifying for a rollover need not be rolled over. (Chapter 24)

152
Q

Which of the following statements concerning funding vehicles in a qualified plan is (are) correct?
I. Annuity contracts can act as the plan’s funding vehicle.
II. A plan can be funded with a common trust fund only if nonplan assets are from other IRA accounts.

A

I only
The answer is (A). II is incorrect because a common trust fund is only allowed if all assets are from qualified plans. (Chapter 11)

153
Q

Loans are prohibited in which qualified plans?

A

SEP, SIMPLE, IRA’s

154
Q

A loan may not be feasible from these plans if the plans do not have nonstock liquid assets?

A

ESOP & Stock bonus plans

155
Q

A loan provision is rarely chosen for these plan types because of administrative complexity?

A

Defined benefit plans and cash-balance plans

156
Q

Cash-balance pension plans

A

The cash-balance plan is often referred to as a hybrid plan because it is a defined-benefit plan that is designed to look like a defined-contribution plan. As a defined-benefit plan, it is subject to minimum funding requirements and the PBGC insurance program. At the same time, the
defined-contribution-like design means an easy-to-explain account balance type of plan. The heart of the cash-balance plan is the benefit structure. As in the defined-contribution plan, the benefit is stated as an account balance that increases with contributions and investment experience. However, in a cash-balance plan the account is hypothetical. Annual credits (referred to as pay credits) are a bookkeeping credit only—no actual contributions are
allocated to participants’ accounts.

157
Q

Target Benefit Plans

A

A target-benefit pension plan identifies a targeted benefit at retirement, and contributions are made in the amount
necessary to fund the targeted amount. The plan specifies the actuarial method and interest rates used to determine annual contributions so that the amount of contribution can be clearly determined.

158
Q

Pension plan vs profit-sharing plans
Is the employer committed to annual funding?
Is there withdrawal flexibility for participants?
Max investment in company stock?

A

Pension Profit sharing
Yes No
No After 2 years
10% max Unlimited